If x = -12, y = -3; find xy² ?​

Answers

Answer 1
Given:-

x = -12

y = -3

To find out:-

Find the value of xy².

Solution:-

xy²

★ Substituting the values of x and y ,we get :

⇒ -12 × ( -3 )²

⇒ -12 × 9

⇒ -108


Related Questions

What is the value of z for the equation fraction 1 over 2z = −fraction 3 over 4 + fraction 1 over 4z? −3 −1 1 3

Answers

Answer:

z= -3

Step-by-step explanation:

Given:

1/2z =-3/4 + 1/4z

Collect like terms

1/2z - 1/4z = -3/4

Add 1/2z - 1/4z

2z-z / 4 = -3/4

We have

z/4=-3/4

Same as

z(1/4) = -3/4

Divide both sides by 1/4

z(1/4) ÷ 1/4 = -3/4÷1/4

z(1/4) × 4/1= -3/4 × 4/1

z(4/4) = -12/4

z= -3

The value of z= -3

Answer:

-3

Step-by-step explanation:

I got it right on the test

In a right angled triangle ABC, ACB =30 and AC=10cm a. calculate BAC b. calculate line AB​

Answers

Answer:

10 cm is the answer because 30÷3 angles

HELP!!!
The solutions to (x + 3)^2- 4=0 are x = -1 and x = -5

True or false

Answers

Answer:

False

Step-by-step explanation:

We can simplify this equation and then solve for x.

[tex](x+3)^3-4=0\\\\x^2+6x+9-4=0\\\\x^2+6x+5=0\\\\(x+2)(x+3)=0\\\\x=-3\\x=-2[/tex]

As you can see, the solutions are not x=-1 and x=-5.

Therefore, the answer is false.

Answer:

True

Step-by-step explanation:

Given

(x + 3)² - 4 = 0 ( add 4 to both sides )

(x + 3)² = 4 ( take the square root of both sides )

x + 3 = ± [tex]\sqrt{4}[/tex] = ± 2 ( subtract 3 from both sides )

x = - 3 ± 2

Thus

x = - 3 - 2 = - 5

x = - 3 + 2 = - 1

Solve logs (8 - 3x) = log20 for x.
A. X = 14
B. X = -13
C.x = -8
D. X= -4

Answers

Answer:

x = -4

Step-by-step explanation:

logs (8 - 3x) = log20

Since we are taking the log on each side

log a = log b  then a = b

8 -3x = 20

Subtract 8  from each side

8 -3x-8 =20 -8

-3x = 12

Divide by -3

-3x/-3 = 12/-3

x = -4

Answer:

[tex] \boxed{\sf x = -4} [/tex]

Step-by-step explanation:

[tex] \sf Solve \: for \: x \: over \: the \: r eal \: numbers:[/tex]

[tex] \sf \implies log(8 - 3x) = log 20[/tex]

[tex] \sf Cancel \: logarithms \: by \: taking \: exp \: of \: both \: sides:[/tex]

[tex] \sf \implies 8 - 3x = 20[/tex]

[tex] \sf Subtract \: 8 \: from \: both \: sides:[/tex]

[tex] \sf \implies 8 - 3x - 8 = 20 - 8 [/tex]

[tex] \sf \implies - 3x = 12 [/tex]

[tex] \sf Divide \: both \: sides \: by \: - 3:[/tex]

[tex] \sf \implies \frac{-3x}{-3} = \frac{12}{-3} [/tex]

[tex] \sf \implies x = - 4[/tex]

x - (-20) = 5 _________________

Answers

X - (-20) = 5

When you subtract a negative, change it to addition:

X + 20 = 5

Subtract 20 from both sides:

X = -15

Answer:

[tex]\boxed{x=-15}[/tex]

Step-by-step explanation:

[tex]x-(-20)=5[/tex]

[tex]\sf Distribute \ negative \ sign.[/tex]

[tex]x+20=5[/tex]

[tex]\sf Subtract \ 20 \ from \ both \ sides.[/tex]

[tex]x+20-20=5-20[/tex]

[tex]x=-15[/tex]

solve for x 5(x+1)=4(x+8)

Answers

Answer:

x=27

Step-by-step explanation:

expanding the above expression we get

5x+5=4x+32

grouping numbers with coefficient of x at the left side and constant at the right side we get

5x-4x=32-5

x=27

To solve this equation, we start by distributing both the 5 and the 4 through both set of parentheses.

This gives us 5x + 5 = 4x + 32.

Now subtract 4x from both
sides to get x + 5 = 32.

Now subtract 5 from both sides and x = 27.

the area of a trapezium is 14.7cmsquare. if the parallel sides are 5.3cm and 3.1cm long,find the perpendicular distance between them

Answers

The perpendicular distance of the trapezoid is 3.5 cm

How to determine the perpendicular distance?

The given parameters are:

Parallel sides = 5.3 cm and 3.1 cmArea = 14.7 square cm

The area of a trapezoid is:

Area = 0.5 * (Sum of parallel sides) * perpendicular distance

So, we have:

14.7 = 0.5 *(5.3 + 3.1) * perpendicular distance

Evaluate

Perpendicular distance = 3.5

Hence, the perpendicular distance of the trapezoid is 3.5 cm

Read more about area at:

https://brainly.com/question/76387

#SPJ1

Which expression is equivalent to (–2)(a + 6)?
A. –2a + 6
B. 2a + 12
C. –2a – 12
D. –2a + 12

Answers

The answer is option c.

There are a total of two hundred students and chaperones going on a
field trip. Each bus can hold 60 passengers. How many buses will be
used for the field trip? Explain why your answer is reasonable.

Answers

Answer:

4 bus is required for field trip to carry 200 passengers.

Step-by-step explanation:

Total no . of passengers = 200

let the be x bus required to carry 200 passengers

capacity of 1 bus = 60 passengers

capacity of x bus = 60*x passengers = 60x passengers

Thus,

60x = 200

x = 200/60 = 3 2/3

thus, 3.66 bus is required , but no. of bus cannot be in fraction hence we take integral value greater than 3.66 which is 4

Thus, 4 bus is required for field trip to carry 200 passenger.

If you invest $600 at 5% interest compounded continuously, how much would you make after 6 years?

Answers

Answer:

809.915$

Step-by-step explanation:

Amount of money = Principal x e^(rate x year)

                              = 600 x e^(0.05 x 6)

                              = 809.915$

Answer:

$809.92

Step-by-step explanation:

(see attached for reference)

Recall that the formula for compound interest (compounded continuously) is

A = P e^(rt)

where,

A = final amount (we are asked to find this)

P = principal = given as $600

r = interest rate = 5% = 0.05

t = time = 6 years

e = 2.71828 (mathematical constant)

Substituting the known values into the equation:

A = P e^(rt)

= 600 e^(0.05 x 6)

= 600 (2.71828)^(0.30)

= $809.92

Barry has been watching the geese that live in his neighborhood. The number of geese changes each week. n f(n) 1 56 2 28 3 14 4 7 Which function best shows the relationship between n and f(n)? f(n) = 28(0.5)n f(n) = 56(0.5)n−1 f(n) = 56(0.5)n f(n) = 112(0.5)n−1

Answers

Answer:

B. f(n) = 56(0.5)^n-1

Step-by-step explanation:

 First, You have to find out the starting population, if you look at the problem you see the population starts at 56

f(x) = 56

Second, you know that the population goes down 50% each week so it has a decay of 0.5

f(x) = 56(0.5)

 Third,  you need to add the exponent of n to make it exponential.  But, if you just add n then the the population would be 28 on week 1 which is incorrect. To fix that you make the exponent n-1 so when you are on week 1 it doesn't become 28 but it stays on 56, and on week 2 it's 28, ect

f(x) = 56(0.5)^n-1

Using a table of values, determine the solution to the equation below to the nearest fourth of a unit. 2^x=1-3^x

Answers

Answer:

Option (1)

Step-by-step explanation:

Given equation is,

[tex]2^x=1-3^x[/tex]

To determine the solution of the equation we will substitute the values of 'x' given in the options,

Option (1)

For x = -0.75

[tex]2^{-0.75}=1-3^{-0.75}[/tex]

0.59 = 1 - 0.44

0.59 = 0.56

Since, values on both the sides are approximately same.

Therefore, x = -0.75 will be the answer.

Option (2)

For x = -1.25

[tex]2^{-1.25}=1-3^{-1.25}[/tex]

0.42 = 1 - 0.25

0.42 = 0.75

Which is not true.

Therefore, x = -1.25 is not the answer.

Option (3)

For x = 0.75

[tex]2^{0.75}=1-3^{0.75}[/tex]

1.68 = 1 - 2.28

1.68 = -1.28

Which is not true.

Therefore, x = 0.75 is not the answer.

Option (4)

For x = 1.25

[tex]2^{1.25}=1-3^{1.25}[/tex]

2.38 = 1 - 3.95

2.38 = -2.95

It's not true.

Therefore, x = 1.25 is not the answer.

What property is demonstrated here? (3x-5) x 4 = 3 x (-5 x 4) A) commutative property of addition B) associative property of multiplication C) commutative property of multiplication D) associative property of addition (haven't learned this yet so I have no clue)

Answers

Answer:

B) Associative Property of Multiplication

Step-by-step explanation:

*if it's wrong idk how, but I apologise*

Which of the following lists of three numbers could form the side lengths of a triangle? A. 10, 20, 30 B. 122, 257, 137 C. 8.6, 12.2, 2.7 D. 1/2, 1/5, 1/6

Answers

Answer:

Step-by-step explanation:

The triangle inequality theorem states that the sum of any two sides of a triangle os greater than the third side.

■■■■■■■■■■■■■■■■■■■■■■■■■■

First triangle:

Let a,b and c be the sides of the triangle:

● a = 10

● b = 20

● c = 30

Now let's apply the theorem.

● a+b = 10+20=30

That's equal to the third side (c=30)

●b+c = 50

That's greater than a.

● a+c = 40

That's greater than b.

These aren't the sides of a triangel since the first inequality isn't verified.

■■■■■■■■■■■■■■■■■■■■■■■■■

Second triangle:

● a = 122

● b = 257

● c = 137

Let's apply the theorem.

● a+b = 379

That's greater than c

● a+c = 259

That's greater than b

● b+c = 394

That's greater than a

So 122,257 and 137 can be sides of a triangle.

■■■■■■■■■■■■■■■■■■■■■■■■■■

The third triangle:

● a = 8.6

● b = 12.2

● c = 2.7

Let's apply the theorem:

● a+b = 20.8

That's greater than c

● b+c = 14.9

That's greater than a

● a+c = 11.3

That isn't greater than b

So theses sides aren't the sides of triangle.

■■■■■■■■■■■■■■■■■■■■■■■■■■

● a = 1/2

● b = 1/5

● c = 1/6

Let's apply the theorem.

● a+b = 7/10

That's greater than c

● a+c = 2/3

That's greater than b

● b+c = 11/30

That isn't greater than a

So these can't be the sides of a triangle.

PLEASE HELPP on THIS PICTURE FOR ONE OF MY QUESTIONS

Answers

Answer:

Linear pair postulate

Step-by-step explanation:

The Linear Pair Postulate states: "If two angles form a linear pair, then the angles are supplementary; that is, the sum of their measures is 180 degrees

A linear pair of angles is such that the sum of angles is 180 degrees.

A car is averaging 50 miles per hour. If the car maintains this speed, how many minutes less would a 450-mile trip take than a 475-mile trip?

Answers

Answer:

1/2 a minute (30 seconds)

Step-by-step explanation:

475/50=9.5

450/50=9

9-9.5=.5

what is the radical of 5√72 PLZ HELP!

Answers

Answer: Exact Form: 30√2

Decimal Form:42.42640687…

Step-by-step explanation: Simplify the radical by breaking the radicand up into a product of known factors, assuming positive real numbers.

I hope this helped :)

Answer:

  30√2

Step-by-step explanation:

The radical portion of the given expression is √72.

__

Perhaps you want the simplest form of your expression. Factor out the perfect squares from under the radical.

  [tex]5\sqrt{72}=5\sqrt{36}\sqrt{2}=5\cdot 6\sqrt{2}=\boxed{30\sqrt{2}}[/tex]

Marta esta poniendo sus libros en una estantería. Le faltan 7 libros para poder poner 12 en cada estante; sin embargo, si pone 10 libros en cada estante, se quedan 5 libros sin poner. ¿Cuantos es antes tiene la estantería?

Answers

Answer:

x = 6       la cantidad de estantes

y = 65    cantidad de libros

Step-by-step explanation:

LLamemos "x" la cantidad de estantes que tiene Marta, y llamaremos "y" la cantidad de libros.

La primera condición que se debe cumplir es que cuando Marta coloca 12 libros en cada estante entonces le faltan 7, esto lo expresamos así:

y  +  7  = 12*x        (1)

La segunda condición establece que si Marta coloca los libros en número de 10 por estante le quedan 5 sin colocar, luego esto en lenguaje matemático se expresa así:

y  -   5   = 10*x     (2)

Ahora hemos obtenido un sistema de dos ecuaciones con dos incógnitas que se resuelve por cualquiera de los métodos conocidos, usaremos el método de sustitución.

Despejamos   y en la primera ecuación y lo sustituimos en la segunda, de esa forma obtendremos el valor de x

y  =  12*x  - 7

(12*x - 7 ) - 5  = 10*x

2*x  -12 = 0

2*x = 12

x = 6       la cantidad de estantes, y

y  =  12*x -7

y  =  72  -  7

y =  65    cantidad de libros

10. RP3-M

Jeanette purchased a concert ticket on a web site. The original price of the ticket was $75.

She used a coupon code to receive a 20% discount. The website applied a 10% service fee

to the discounted price. Jeannette's ticket was less than the original price by what percent?

Answers

Answer:

Jeannette's ticket was less than the original pice by 30%

Step-by-step explanation:

original price = $75

percentage discount = 20% of original price = 20% of $75

discounted price = [tex]\frac{20}{100} \times\ 75\ =\ 15[/tex]

discounted price = $15

website service fee = 10% of original price

website service fee = [tex]\frac{10}{100}\times 75 = \$7.5[/tex]

New discounted price = discount price + website service fee

= 15 + 7.5 = $22.5

Next, let us calculate what percentage of the original price that will give the new discount price.

Let the percentage of the original price = x%

x% of 75 = $22.5

[tex]\frac{x}{100} \times\ 75\ = 22.5\\\\\frac{75x}{100} = 22.5\\\\75x = 2250\\\\x = \frac{2250}{75} \\\\x = 30[/tex]

Therefore, Jeannette's ticket was less than the original pice by 30%

help..? why are there so many parentheses..?can you plz give a step by step on how to slove the equation?

Answers

Answer:

= -11

Step-by-step explanation:

-(-(11-22))

= -(-11+22)

= 11 - 22

= -11

Point E is on line segment DF. Given DE=9 and DF=11, determine the length EF.

Answers

Answer: Line EF=2

Step-by-step explanation: 11 minus 9 is equal to 2. So line EF is equal to 2.

Lenny is competing with his cousin, Jasper, in an indoor rock-climbing contest. At the start of the climb, Lenny makes his way 5 ¼ feet up the wall, while Jasper climbs 9 ¾ feet. How much farther did Jasper climb than Lenny?

Answers

Answer:

[tex]4\frac{1}{2}[/tex] feet further.

Step-by-step explanation:

Since these are mixed numbers that are both in fourths, we can easily subtract the two numbers. However, I find it easier if we first convert both mixed numbers into improper fractions.

[tex]5\frac{1}{4} = \frac{5\cdot4+1}{4} = \frac{21}{4}[/tex]

[tex]9\frac{3}{4} = \frac{9\cdot4+3}{4} = \frac{39}{4}[/tex]

Now we can subtract the numerators:

[tex]\frac{39}{4} - \frac{21}{4} = \frac{39-21}{4} = \frac{18}{4}[/tex]

[tex]\frac{18}{4}[/tex] simplifies down to [tex]\frac{9}{2}[/tex].

Converting  [tex]\frac{9}{2}[/tex] to a mixed number is easy - 2 goes into 9 4 times (8) with one remainder so:

[tex]4\frac{1}{2}[/tex] .

Hope this helped!

For the function F(x)= 1/x-2 whose graph is shown below, what is the relative value of F(x) when the value of x is close to 2?​

Answers

Answer:

10,000

Step-by-step explanation:

**Yoxelt buys 4 1/ 2 gallons of soda. One-fourth of the soda he bought was Pepsi and the rest was Sprite. How many gallons of Pepsi did Yoxelt buy? Show all work below.

Answers

Answer:

1 1/8

Step-by-step explanation:

1/4 of 4 1/2 is Pepsi.

1/4 * 4 1/2 = (1/4) * 4 + (1/4) * (1/2) = 1 1/8

BRAINLIEST, 5 STARS AND THANKS IF ANSWERED CORRECTLY.

A quadratic equation with a negative discriminant has a graph that..

A. touches the x-axis but does not cross it
B. opens downward and crosses the x-axis twice
C. crosses the x-axis twice.
D. never crosses the x-axis.

Answers

Answer:

never crosses the x-axis.

Step-by-step explanation:

     A quadratic equation with a negative discriminant has a graph that - never crosses the x-axis.

Answer:

The graph of a quadratic equation that has a negative discriminant is the one that never intersect x-axis. The graph of a quadratic equation that has a zero discriminant is the one that intersect x-axis at only one point. To be clearer, it can be seen in the attached image.

Step-by-step explanation:

Answer D

Given the right triangle below, if AB = 4 and BC = 4, find AC.
A
B
C

Answers

The length of AC is found using the Pythagorean theorem which is a^2+b^2=c^2.

In this case your equation would look like 4^2+4^2=c^2.
Four to the power of two is 16 so 16+16=c^2.
16+16=32
32=c^2
Root 32 = c

Length AC is root 32.

AC will be 4√2 when AB = 4 and BC = 4, in the given right triangle.

What is Pythagoras' Theorem?

According to Pythagoras' Theorem, in a right triangle, the square of the length of the longest side, that is, the hypotenuse, that is, the side opposite to the right angle is equal to the sum of the squares of the lengths of the other two sides.

How to solve the question?

In the question, we are given a right triangle, with sides AB = 4 and BC = 4.

We are asked to find AC.

To find AC, we will use the Pythagoras theorem, according to which, we can write:

AC² = AB² + BC²

or, AC² = 4² + 4²,

or, AC² = 16 + 16,

or, AC² = 32,

or, AC = √32,

or, AC = √(16 * 2) = 4√2.

Therefore, AC will be 4√2 when AB = 4 and BC = 4, in the given right triangle.

Learn more about Pythagoras' Theorem at

https://brainly.com/question/231802

#SPJ2

What are the domain and range of the real-valued function f(x)=2/(x+5)?

Answers

Answer:

Domain is all real numbers, x ≠ -5

Range is all real numbers, y ≠ 0

Step-by-step explanation:

Domain: all real numbers except x=-5
(-infinity, -5)U(-5, infinity)

Range: all real numbers except y=0
(-infinity, 0)U(0, infinity)

Simplify the expression. (3x2 – 4x + 1) + (-x2 + x – 9)

Answers

[tex](3x^2 - 4x + 1) + (-x^2 + x - 9)=\\3x^2-4x+1-x^2+x-9=\\2x^2-3x-8[/tex]

= (6x+4x+1)+(-2x+x-9)
= (2x+1)+(-x-9)
= 2x+1-x-9
= x-8

The row-echelon form of the augmented matrix of a system of equations is given.Find the solution of the system

Answers

Answer:

x = 9/4

y = 3/5

z = 2/3

w = -9/5

Step-by-step explanation:

Technically, the matrix is in reduced row echelon form. If there are zeros above and below the ones, it is RREF. If there are zeros only below the ones, then it's REF.

Since it is in RREF, the augmented numbers to the right of the bar are already your solutions. Simply label the variables.

Allison is rolling her hula hoop on the playground. The radius of her hula hoop is 35 \text{ cm}35 cm35, start text, space, c, m, end text. What is the distance the hula hoop rolls in 444 full rotations?

Answers

Answer: 880 cm

Step-by-step explanation:

Given: Radius of the hula hoop = 35 cm

Hula hoop is  circular in shape

Then, Circumference = [tex]2\pi r[/tex] , where r = radius

Now , Circumference of hula hoop = [tex]2\times \dfrac{22}{7}\times35=220\ cm[/tex]

Now , the distance the hula hoop rolls in 4 full rotations = 4 × (Circumference of hula hoop)

[tex]= 4 \times 220=880\ cm[/tex]

Hence, the required distance = 880 cm

Answer:

880

Step-by-step explanation:

Other Questions
3. What are the first steps that you should take if you are unable to get onto the Internet? (1 point)O Check your router connections then restart your router.O Plug the CPU to a power source and reboot the computer.O Adjust the display properties and check the resolution.Use the Control Panel to adjust the router settings. For a sample of eight bears, researchers measured the distances around the bears' chests and weighed the bears. Minitab was used to find that the value of the linear correlation coefficient is r=0.989. Using alph=0.05, determine if there is a linear correlation between chest size and weight. What proportion of the variation in weight can be explained by the linear relationship between weight and chest size? HighLife Corporation has the following information: Average demand = 30 units per day Average lead time = 40 days Item unit cost = $45 for orders of less than 400 units Item unit cost = $40 for orders of 400 units or more Ordering cost = $50 Inventory carrying cost = 15 percent The business year is 300 days. Standard deviation of demand during lead time = 90 Desired service level = 95 percent What is the EOQ if HighLife pays $45/unit? Due to possible differences in rounding, choose the closest answer.\ The risk-free rate of return is 3.2 percent and the market risk premium is 4.6 percent. What is the expected rate of return on a stock with a beta of 2.12 Quadratic and Ratios... When did the Aztec civilisation Rise 7(x+1)=21 solve for x Which of the functions below could have created this graph?O A. F(x) = -x' +5x +7O B. F(x) = 2x2 - 4x2 +4O C. F(x)=x2+x+3O D. F(x) = -5x 2x+5 HELP ASAP HELP ASAP HELP ASAP HELP ASAP HELP ASAP HELP ASAP HELP ASAP HELP ASAP HELP ASAP Which expression is equivalent to 5y^3/(5y)^-2 check to see whether 5 is a solution: 10 + 7g < 44 Find the sum of 29,the product of 2 and 9 and the difference between 2 and 9 15 points! I will give Brainliest and heart! Answer ASAP but with DETAIL, I need step - by - step, clear words, correct grammar. A pair of equations is shown below: y = 3x 5 y = 6x 8 Part A: Explain how you will solve the pair of equations by substitution or elimination. Show all the steps and write the solution. (5 points) Part B: If the two equations are graphed, at what point will the lines representing the two equations intersect? Explain your answer. (5 points) write an equation of the perpendicular bisector of the segment joining a(-2,3) and b(4,-5).A) 3x+4y=7 B) 3x-4y=-7 C) 3x-4y=7 D) -3x-4y=7 E) 4x-3y=7 Which is a focus of American Reform literature?-the evil of humans-the effect of intuition-the need for imagination-the importance of human rights A, B, C, D, E, F, ... 2, 3, 5, 7, 11, 13, ... what number is the letter Z replaced with? The fact that the top causes of death in the United States are due to chronic illness while people in the developing world are continually affected by the threat of acute illnesses indicates that Question 2(Multiple Choice Worth 2 points)(01.07 MC)Read the incomplete sentence and choose the option with the correct verbs to complete it.las dos de la maana. Seor Federico, bien?O Es, estoyO Soy, estsO Somos estnSon, est "Tough Guys 2," the traditional way of looking at violence as something that is_________, should be augmented with a new view that emphasizes that violence is something _________.. Explain how you would value a stock. Provide an example of a valuation of a stock based on retrieved real data. Include evidence of the retrieved data in your answer. Compare your valuation with the actual price of the stock at the designated time for your valuation.